Netmedicos.com

Turning Dreams of Medicos Into Reality...!!!
Mcqs MBBS Exams India
Welcome, GuestLogin / Register
Home >> Exams >> AIPGMEE >> AIPGMEE 2006 >> Mcqs Page 19
(Single Best Answer) Question 181: Which of the following ovarian tumor is most prone to undergo torsion during pregnancy ?
A) Serous Cystadenoma
B) Mucenous Cystadenoma
C) Dermoid cyst
D) Theca Lutein cyst
Answer (Select an option above to get the answer):
(Single Best Answer) Question 182: Which one of the following congenital malformation of the fetus can be diagnosed in first trimester by ultrasound ?
A) Anencephaly
B) Inencephaly
C) Microcephaly
D) Holoprosencephaly
Answer (Select an option above to get the answer):
(Single Best Answer) Question 183: The most common cause of tubal block in India is ?
A) Gonorrhoea infection
B) Chlamydia infection
C) Tuberculosis
D) Bacterial vaginosis
Answer (Select an option above to get the answer):
(Single Best Answer) Question 184: The following complications during pregnancy increase the risk of Post Partum Haemorrhage (PPH) except ?
A) Hypertension
B) Macrosomia
C) Twin pregnancy
D) Hydramnios
Answer (Select an option above to get the answer):
(Single Best Answer) Question 185: Indicators of impending uterine rupture during labour include all of the following except ?
A) Fetal distress
B) Hematuria
C) Fresh bleeding per vaginum
D) Passage of meconium
Answer (Select an option above to get the answer):
(Single Best Answer) Question 186: Which of the following statements is incorrect regarding polycystic ovarian disease ?
A) Elevated LH hormone
B) Can cause infertility
C) May be associated with abnormal glucose tolerance test
D) Results in postdated pregnancy
Answer (Select an option above to get the answer):
(Single Best Answer) Question 187: Which of the following statements is incorrect regarding levonorgestral releasing intra-uterine system ?
A) There is increased incidence of menorrhagia
B) This system can be used as hormone replacement therapy
C) This method is useful for the treatment of Endometrial hyperplasia
D) Irregular uterine bleeding can be a problem initially
Answer (Select an option above to get the answer):
(Single Best Answer) Question 188: All of the following mechanisms might account for a reduced risk of upper genital tract infection in users of progestin-releasing IUDs, except ?
A) Reduced retrograde menstruation
B) Decreased ovulation
C) Thickened cervical mucus
D) Decidual changes in the endometrium
Answer (Select an option above to get the answer):
(Single Best Answer) Question 189: Emergency contraception prevents pregnancy by all of the following mechanisms, except ?
A) Delaying / inhibiting ovulation
B) Inhibiting fertilization
C) Preventing implantation of the fertilized egg
D) Interrupting an early pregnancy
Answer (Select an option above to get the answer):
(Single Best Answer) Question 190: Misoprostol is a ?
A) Prostaglandin E1 analogue
B) Prostaglandin E2 analogue
C) Prostaglandin antagonist
D) Antiprogestin
Answer (Select an option above to get the answer):